Consumer magazine: Because front-loading washers use less water than top-loading washers, ordinary powder detergent d...

Jaimee-Salgado on October 17, 2018

Can you please explain?

Can someone explain why D is the correct answer and not C?

Reply
Create a free account to read and take part in forum discussions.

Already have an account? log in

Mehran on October 17, 2018

Hi @Jaimee-Salgado, thanks for your post. Let's start, as always, with the stimulus. This one presents an argument. The conclusion is: "to get clothes really clean in a front-loading machine you need to use a detergent formulated especially for front-loading washers, instead of ordinary powder detergent."

What premise is given in support of this conclusion? All you are told is that ordinary powder detergent does not dissolve readily in front-loading washers.

Notice the gap in the reasoning. The premise is about whether powder detergent readily dissolves in front-loading washers. But the conclusion is about getting clothes really clean. The two ideas - dissolving detergent and clean clothes - are not strongly linked.

The question stem asks you to identify the necessary assumption that would strengthen the argument (bridge this gap). Answer choice (D) does this - it connects the concept of dissolving detergent with the concept of really clean clothes.

Answer choice (C) does not address the gap in the argument. Rather, (C) seems to restate the given conclusion itself, rather than addressing the premise and the conclusion together. If you added this idea to the stimulus, it would start to feel circular: "you need to use specially formulated detergent to get clothes really clean to get clothes really clean."

Remember, too, that negating the correct answer on a Strengthen with Necessary Premise question would make the argument in the stimulus fall apart. Let's negate the correct answer, (D): "A laundry detergent might get clothes really clean in a washer even if it does not dissolve readily in it." If this is true, the argument in the stimulus falls apart, does it not? That's how you know you have the right answer.

Hope this helps! Please let us know if you have any additional questions.